Manufacturers sometimes discount the price of a product to

This topic has expert replies
Legendary Member
Posts: 944
Joined: Wed May 30, 2012 8:21 am
Thanked: 8 times
Followed by:5 members
Manufacturers sometimes discount the price of a product to retailers for a promotion period when the product is advertised to consumers. Such promotions often result in a dramatic increase in amount of product sold by the manufacturers to retailers. Nevertheless, the manufacturers could often make more profit by not holding the promotions.

Which of the following, if true, most strongly supports the claim above about the manufacturers' profit?

(A) The amount of discount generally offered by manufacturers to retailers is carefully calculated to represent the minimum needed to draw consumers' attention to the product.
(B) For many consumer products the period of advertising discounted prices to consumers is about a week, not sufficiently long for consumers to become used to the sale price.
(C) For products that are not newly introduced, the purpose of such promotions is to keep the products in the minds of consumers and to attract consumers who are currently using competing products.
(D) During such a promotion retailers tend to accumulate in their warehouses inventory bought at discount; they then sell much of it later at their regular price.
(E) If a manufacturer falls to offer such promotions but its competitor offers them, that competitor will tend to attract consumers away from the manufacturer's product.


OA : D

P.S: @ Verbal Experts - It took me around 3.5 minutes to solve this...Could you please share your DETAILED analysis and explanation for this Qs ? Also, it'd be great if you could please shed some light on BIT faster solution!

User avatar
Legendary Member
Posts: 2131
Joined: Mon Feb 03, 2014 9:26 am
Location: https://martymurraycoaching.com/
Thanked: 955 times
Followed by:140 members
GMAT Score:800

by MartyMurray » Tue Oct 06, 2015 10:50 pm
Maybe the key to quickly and accurately getting the answer to this one is noticing exactly what is being said in the prompt.

The prompt says that during the promotional period manufacturers see large increases in the amount of product sold. However they would be more profitable if they did not use the promotions.

So the question to be answered is the following. Why is using promotional sales and experiencing increased sales less profitable than not doing so?

A) This seems to be a reason why the holding promotions would be more profitable rather than less profitable than not doing so. So this isn't it.

B) This supports the idea that promotional periods would not affect consumer habits in a way that would hurt profits. So this isn't it.

C) This discusses the purpose of the promotions and does not provide anything that explains why holding them reduces profits.

D) What this implies is that much of the increase in sales seen during promotional periods is the result of retailers' loading up on discounted product that they will sell later. So what's going on is that instead of selling at discounted prices only product that will be sold during the promotional period, the manufacturers are selling at discounted prices product that will be sold both during and after the promotional period. This could hurt their profits if some of the sales of discounted product are replacing some sales of fully priced product. So this is probably the OA.

E) This is a reason to engage in the promotions and does provide information that helps us understand why holding promotions is less profitable than not doing so.

So D is definitely the best answer.
Marty Murray
Perfect Scoring Tutor With Over a Decade of Experience
MartyMurrayCoaching.com
Contact me at [email protected] for a free consultation.

User avatar
GMAT Instructor
Posts: 15539
Joined: Tue May 25, 2010 12:04 pm
Location: New York, NY
Thanked: 13060 times
Followed by:1906 members
GMAT Score:790

by GMATGuruNY » Thu Oct 08, 2015 6:08 am
RBBmba@2014 wrote:Manufacturers sometimes discount the price of a product to retailers for a promotion period when the product is advertised to consumers. Such promotions often result in a dramatic increase in amount of product sold by the manufacturers to retailers. Nevertheless, the manufacturers could often make more profit by not holding the promotions.

Which of the following, if true, most strongly supports the claim above about the manufacturers' profit?

(A) The amount of discount generally offered by manufacturers to retailers is carefully calculated to represent the minimum needed to draw consumers' attention to the product.
(B) For many consumer products the period of advertising discounted prices to consumers is about a week, not sufficiently long for consumers to become used to the sale price.
(C) For products that are not newly introduced, the purpose of such promotions is to keep the products in the minds of consumers and to attract consumers who are currently using competing products.
(D) During such a promotion retailers tend to accumulate in their warehouses inventory bought at discount; they then sell much of it later at their regular price.
(E) If a manufacturer falls to offer such promotions but its competitor offers them, that competitor will tend to attract consumers away from the manufacturer's product.


OA : D

P.S: @ Verbal Experts - It took me around 3.5 minutes to solve this...Could you please share your DETAILED analysis and explanation for this Qs ? Also, it'd be great if you could please shed some light on BIT faster solution!
Premise: Such promotions often result in a dramatic increase in amount of product sold by the manufacturers to retailers.
Conclusion: Manufacturers could often make more profit by not holding the promotions.

To strengthen the conclusion that promotions hurt profit margins, the correct answer must BREAK THE LINK between the AMOUNT OF PRODUCT SOLD TO RETAILERS and the AMOUNT OF PROFIT EARNED BY MANUFACTURERS.

D: During such a promotion retailers tend to accumulate in their warehouses inventory bought at discount; they then sell much of it later at their regular price.
Let's say that the promotions are held only in the winter.
D implies the following:
Instead of purchasing product in the spring, summer and fall seasons at the REGULAR PRICE, retailers purchase product for the whole year at the DISCOUNTED WINTER PRICE, resulting in a decrease in manufacturers' annual revenues and supporting the conclusion that manufacturers could often make more profit by not holding the promotions.

The correct answer is D.
Private tutor exclusively for the GMAT and GRE, with over 20 years of experience.
Followed here and elsewhere by over 1900 test-takers.
I have worked with students based in the US, Australia, Taiwan, China, Tajikistan, Kuwait, Saudi Arabia -- a long list of countries.
My students have been admitted to HBS, CBS, Tuck, Yale, Stern, Fuqua -- a long list of top programs.

As a tutor, I don't simply teach you how I would approach problems.
I unlock the best way for YOU to solve problems.

For more information, please email me (Mitch Hunt) at [email protected].
Student Review #1
Student Review #2
Student Review #3

Legendary Member
Posts: 944
Joined: Wed May 30, 2012 8:21 am
Thanked: 8 times
Followed by:5 members

by RBBmba@2014 » Fri Oct 09, 2015 9:59 am
GMATGuruNY wrote:D implies the following:
Instead of purchasing product in the spring, summer and fall seasons at the REGULAR PRICE, retailers purchase product for the whole year at the DISCOUNTED WINTER PRICE...
Hi GMATGuruNY,
(1) Got a doubt on your above quote(re RED part). How we can be sure about that ? Couldn't it be the case that MUCH (BUT NOT ALL) of the products sold by retailers during non-promotion period was purchased from the manufacturers during promotion.

Please clarify.

(2) Could you please let me know whether my following understanding is correct ?

The OA seems to imply that the retailers purchase products from the manufacturers at DISCOUNTED PRICE during promotions in such a QUANTITY that the retailers sell much of this UNSOLD stocks to the consumers during non-promotion period (at non-DISCOUNTED/REGULAR PRICE),either NOT buying AT ALL from the manufacturers during this non-promotion period or buying further in a much limited quantity than USUAL (as they ALREADY have unsold stocks) . Thus the manufacturers' sales, hence revenue comes down during this non-promotion period than USUAL.

So,BOTTOM-LINE of the OA is (while INPUT COSTS remain CONSTANT), the TOTAL SALES of manufacturers to retailers say,in a year when there is BOTH PROMOTIONAL and NON-PROMOTIONAL PERIOD will be LESS than the TOTAL SALES in a year when there is ONLY NON-PROMOTIONAL PERIOD.

Correct me please if wrong!

(I think, if it were an ASSUMPTION CR then this Option D would not be the OA because of MUST BE TRUE aspect. Thoughts ?)

(3) As for Option B: I think,it neither strengthens nor weakens the conclusion as it doesn't explicitly indicate whether the sales without promotion will be higher/less than the sales with promotion! Thoughts ?

User avatar
GMAT Instructor
Posts: 15539
Joined: Tue May 25, 2010 12:04 pm
Location: New York, NY
Thanked: 13060 times
Followed by:1906 members
GMAT Score:790

by GMATGuruNY » Sat Oct 10, 2015 2:46 am
RBBmba@2014 wrote:
GMATGuruNY wrote:D implies the following:
Instead of purchasing product in the spring, summer and fall seasons at the REGULAR PRICE, retailers purchase product for the whole year at the DISCOUNTED WINTER PRICE...
Hi GMATGuruNY,
(1) Got a doubt on your above quote(re RED part). How we can be sure about that ? Couldn't it be the case that MUCH (BUT NOT ALL) of the products sold by retailers during non-promotion period was purchased from the manufacturers during promotion.

Please clarify.
In my post above, I offered an extreme case to make the reasoning easier to see.
The point of the OA is that a promotion incentivizes retailers to buy as much product as possible at the discounted price -- including product that they would normally buy later in the year at the regular price.
As a result, manufacturers reap less profit.
(2) Could you please let me know whether my following understanding is correct ?

The OA seems to imply that the retailers purchase products from the manufacturers at DISCOUNTED PRICE during promotions in such a QUANTITY that the retailers sell much of this UNSOLD stocks to the consumers during non-promotion period (at non-DISCOUNTED/REGULAR PRICE),either NOT buying AT ALL from the manufacturers during this non-promotion period or buying further in a much limited quantity than USUAL (as they ALREADY have unsold stocks) . Thus the manufacturers' sales, hence revenue comes down during this non-promotion period than USUAL.
Your understanding seems correct.
(I think, if it were an ASSUMPTION CR then this Option D would not be the OA because of MUST BE TRUE aspect. Thoughts ?)
Correct.
Since the OA does not have to be true for the conclusion to be valid, it does not constitute an assumption.
(3) As for Option B: I think,it neither strengthens nor weakens the conclusion as it doesn't explicitly indicate whether the sales without promotion will be higher/less than the sales with promotion! Thoughts ?
It is given as a PREMISE that the promotions often result in a dramatic increase in amount of product sold by the manufacturers to retailers.
B does not break the link between this dramatic increase and the amount of profit earned by manufacturers.
Private tutor exclusively for the GMAT and GRE, with over 20 years of experience.
Followed here and elsewhere by over 1900 test-takers.
I have worked with students based in the US, Australia, Taiwan, China, Tajikistan, Kuwait, Saudi Arabia -- a long list of countries.
My students have been admitted to HBS, CBS, Tuck, Yale, Stern, Fuqua -- a long list of top programs.

As a tutor, I don't simply teach you how I would approach problems.
I unlock the best way for YOU to solve problems.

For more information, please email me (Mitch Hunt) at [email protected].
Student Review #1
Student Review #2
Student Review #3

Legendary Member
Posts: 944
Joined: Wed May 30, 2012 8:21 am
Thanked: 8 times
Followed by:5 members

by RBBmba@2014 » Sat Oct 10, 2015 10:04 am
Hi GMATGuruNY,
All good. Just couple of clarifications on your reply -
BOTTOM-LINE of the OA is (while INPUT COSTS remain CONSTANT), the TOTAL SALES of manufacturers to retailers say,in a year when there is BOTH PROMOTIONAL and NON-PROMOTIONAL PERIOD will be LESS than the TOTAL SALES in a year when there is ONLY NON-PROMOTIONAL PERIOD.
Could you please shed some light whether my above understanding is also correct ?
GMATGuruNY wrote:
(3) As for Option B: I think,it neither strengthens nor weakens the conclusion as it doesn't explicitly indicate whether the sales without promotion will be higher/less than the sales with promotion! Thoughts ?
It is given as a PREMISE that the promotions often result in a dramatic increase in amount of product sold by the manufacturers to retailers.
B does not break the link between this dramatic increase and the amount of profit earned by manufacturers.
Got this.
I guess,another issue with B is it implies that consumers would still continue to purchase products from retailers during the non-promotion period, BUT doesn't explicitly indicate whether the that sales would be higher/less than the USUAL sales during the non-promotion period -- this seems to be a key aspect in determining the above link your mentioned.

Correct me please if wrong!

Master | Next Rank: 500 Posts
Posts: 152
Joined: Fri Apr 24, 2015 1:39 am
Location: Rourkela Odisha India
Thanked: 2 times
Followed by:3 members
GMAT Score:650

by akash singhal » Tue Oct 20, 2015 12:14 am
RBBmba@2014 wrote:Hi GMATGuruNY,
All good. Just couple of clarifications on your reply -
BOTTOM-LINE of the OA is (while INPUT COSTS remain CONSTANT), the TOTAL SALES of manufacturers to retailers say,in a year when there is BOTH PROMOTIONAL and NON-PROMOTIONAL PERIOD will be LESS than the TOTAL SALES in a year when there is ONLY NON-PROMOTIONAL PERIOD.
Could you please shed some light whether my above understanding is also correct ?
GMATGuruNY wrote:
(3) As for Option B: I think,it neither strengthens nor weakens the conclusion as it doesn't explicitly indicate whether the sales without promotion will be higher/less than the sales with promotion! Thoughts ?
It is given as a PREMISE that the promotions often result in a dramatic increase in amount of product sold by the manufacturers to retailers.
B does not break the link between this dramatic increase and the amount of profit earned by manufacturers.
Got this.
I guess,another issue with B is it implies that consumers would still continue to purchase products from retailers during the non-promotion period, BUT doesn't explicitly indicate whether the that sales would be higher/less than the USUAL sales during the non-promotion period -- this seems to be a key aspect in determining the above link your mentioned.

Correct me please if wrong!

Hi,I think you got the concept wrong.here sales are not the point of concern they may be more or equal or less.we are concerned about their profit margin.

For ex. If i am buying 500 boxes from manufactures in the non-promotional period throughtout at 10$ profit at each box for manufactures,then the profit for manufactures is 5000$

Now lets say in promotional period I buy 300 boxes at a 5$ profit from the manufactures and another additional 200 boxes at 10$ profit in non-promotional period. As you can see that the sale figure in both cases are same but profits for 2nd case is less.So the manufactures are better off without discount.

Also if sale is higher at promotional period still profits can be more during non- promotional period.
I think this should clear your doubt.
Do argue if you have any doubt.

Legendary Member
Posts: 944
Joined: Wed May 30, 2012 8:21 am
Thanked: 8 times
Followed by:5 members

by RBBmba@2014 » Tue Oct 20, 2015 9:39 pm
Hi GMATGuruNY - it'd be really helpful if you could please share your thoughts on my above concerns.

Look forward to hear from you Sir...Much thanks in advance!

P.S: FYQR - https://www.beatthegmat.com/manufacturer ... tml#757588

User avatar
GMAT Instructor
Posts: 15539
Joined: Tue May 25, 2010 12:04 pm
Location: New York, NY
Thanked: 13060 times
Followed by:1906 members
GMAT Score:790

by GMATGuruNY » Thu Oct 22, 2015 3:52 am
RBBmba@2014 wrote:BOTTOM-LINE of the OA is (while INPUT COSTS remain CONSTANT), the TOTAL SALES of manufacturers to retailers say,in a year when there is BOTH PROMOTIONAL and NON-PROMOTIONAL PERIOD will be LESS than the TOTAL SALES in a year when there is ONLY NON-PROMOTIONAL PERIOD.
Not quite.
The OA does not imply that the promotion will decrease the NUMBER of units sold to retailers.
It simply implies that retailers will PAY LESS for the purchased units.
Let's say a retailer typically purchases 10 units each month, for a total of 100 units per 10-month period.
The OA implies the following:
Instead of purchasing 10 units at the sale price and 90 units at the regular price, a retailer will purchase all 100 units at the sale price.
While the number of units purchased remains constant, the PRICE paid for these units is lower.
As a result, the manufacturer earns less profit.

I guess,another issue with B is it implies that consumers would still continue to purchase products from retailers during the non-promotion period, BUT doesn't explicitly indicate whether the that sales would be higher/less than the USUAL sales during the non-promotion period -- this seems to be a key aspect in determining the above link your mentioned.
This line of reasoning seems valid.
Private tutor exclusively for the GMAT and GRE, with over 20 years of experience.
Followed here and elsewhere by over 1900 test-takers.
I have worked with students based in the US, Australia, Taiwan, China, Tajikistan, Kuwait, Saudi Arabia -- a long list of countries.
My students have been admitted to HBS, CBS, Tuck, Yale, Stern, Fuqua -- a long list of top programs.

As a tutor, I don't simply teach you how I would approach problems.
I unlock the best way for YOU to solve problems.

For more information, please email me (Mitch Hunt) at [email protected].
Student Review #1
Student Review #2
Student Review #3

Legendary Member
Posts: 944
Joined: Wed May 30, 2012 8:21 am
Thanked: 8 times
Followed by:5 members

by RBBmba@2014 » Sat Oct 24, 2015 4:02 am
GMATGuruNY wrote:
RBBmba@2014 wrote:BOTTOM-LINE of the OA is (while INPUT COSTS remain CONSTANT), the TOTAL SALES of manufacturers to retailers say,in a year when there is BOTH PROMOTIONAL and NON-PROMOTIONAL PERIOD will be LESS than the TOTAL SALES in a year when there is ONLY NON-PROMOTIONAL PERIOD.
Not quite.
The OA does not imply that the promotion will decrease the NUMBER of units sold to retailers.
It simply implies that retailers will PAY LESS for the purchased units.
Let's say a retailer typically purchases 10 units each month, for a total of 100 units per 10-month period.
The OA implies the following:
Instead of purchasing 10 units at the sale price and 90 units at the regular price, a retailer will purchase all 100 units at the sale price.
While the number of units purchased remains constant, the PRICE paid for these units is lower.
As a result, the manufacturer earns less profit.
ABSOLUTELY. In fact, I mentioned about your reasoning in my earlier post, which you already approved. So, no issue with this part...

However,I'd try to clarify what I wanted to convey in my above quote. By TOTAL SALES, I meant Revenue in my above quote -- I didn't mean NUMBER of units sold to retailers. While ALL other relative factors (such as INPUT COSTS/EXPENSES, CONSUMERS' DEMAND) remain constant, increase in Revenue means increase in profit and decrease in Revenue means decrease in profit. Right ?

As this is TRUE then can we really DISCARD the logic I raised in the above quote ? (after all, MUST BE TRUE aspect doesn't hold good for this CR).

Would be curious to know your thoughts.